Laplace transforms - second shift theorem












1














pic 1



pic 2



Hi, i'm struggling with understanding how this expansion came about for determining the LT of f(t). Is anyone familiar with this?
thank you










share|cite|improve this question






















  • Are you unclear with where the functions that they are applying the Laplace transform are coming from, or the actual step in applying the transform?
    – Theo C.
    Jan 4 at 2:06










  • Hi, thanks for your reply. I know the second shift theorem for the unit step function (u) is being applied but I don't understand where the expansions come from.
    – user564900
    Jan 4 at 2:08
















1














pic 1



pic 2



Hi, i'm struggling with understanding how this expansion came about for determining the LT of f(t). Is anyone familiar with this?
thank you










share|cite|improve this question






















  • Are you unclear with where the functions that they are applying the Laplace transform are coming from, or the actual step in applying the transform?
    – Theo C.
    Jan 4 at 2:06










  • Hi, thanks for your reply. I know the second shift theorem for the unit step function (u) is being applied but I don't understand where the expansions come from.
    – user564900
    Jan 4 at 2:08














1












1








1







pic 1



pic 2



Hi, i'm struggling with understanding how this expansion came about for determining the LT of f(t). Is anyone familiar with this?
thank you










share|cite|improve this question













pic 1



pic 2



Hi, i'm struggling with understanding how this expansion came about for determining the LT of f(t). Is anyone familiar with this?
thank you







laplace-transform






share|cite|improve this question













share|cite|improve this question











share|cite|improve this question




share|cite|improve this question










asked Jan 4 at 1:06









user564900

305




305












  • Are you unclear with where the functions that they are applying the Laplace transform are coming from, or the actual step in applying the transform?
    – Theo C.
    Jan 4 at 2:06










  • Hi, thanks for your reply. I know the second shift theorem for the unit step function (u) is being applied but I don't understand where the expansions come from.
    – user564900
    Jan 4 at 2:08


















  • Are you unclear with where the functions that they are applying the Laplace transform are coming from, or the actual step in applying the transform?
    – Theo C.
    Jan 4 at 2:06










  • Hi, thanks for your reply. I know the second shift theorem for the unit step function (u) is being applied but I don't understand where the expansions come from.
    – user564900
    Jan 4 at 2:08
















Are you unclear with where the functions that they are applying the Laplace transform are coming from, or the actual step in applying the transform?
– Theo C.
Jan 4 at 2:06




Are you unclear with where the functions that they are applying the Laplace transform are coming from, or the actual step in applying the transform?
– Theo C.
Jan 4 at 2:06












Hi, thanks for your reply. I know the second shift theorem for the unit step function (u) is being applied but I don't understand where the expansions come from.
– user564900
Jan 4 at 2:08




Hi, thanks for your reply. I know the second shift theorem for the unit step function (u) is being applied but I don't understand where the expansions come from.
– user564900
Jan 4 at 2:08










1 Answer
1






active

oldest

votes


















1














If I've understood your comment correctly, then I think I see the confusion.



Recall that the second shifting theorem says that if $mathcal{L}{ f(t)} = F(s)$ then $mathcal{L}{ f(t-a)u(t-a)} = e^{-as}F(s)$



Now, let's dissect taking the Laplace transform of $frac{1}{2}t^2u(t-1)$. Note that our current function is $f(t) = frac{1}{2}t^2$. So then $f(t-1) = frac{1}{2}(t-1)^2$. As such, to get $f(t)$ in the desired form, we will conveniently add $0$ in a number of different ways:
$$f(t)=f(t) + (t-t) + (frac{1}{2}-frac{1}{2}) = frac{1}{2}(t^2-2t+1) + t - frac{1}{2} = f(t-1) + t-frac{1}{2} + (frac{1}{2}-frac{1}{2}) $$
$$= f(t-1) + (t-1) + frac{1}{2} $$
Now we have $f(t)$ in the desired form to apply the shifting theorem.






share|cite|improve this answer





















  • Thank you!! this was very helpful
    – user564900
    2 days ago











Your Answer





StackExchange.ifUsing("editor", function () {
return StackExchange.using("mathjaxEditing", function () {
StackExchange.MarkdownEditor.creationCallbacks.add(function (editor, postfix) {
StackExchange.mathjaxEditing.prepareWmdForMathJax(editor, postfix, [["$", "$"], ["\\(","\\)"]]);
});
});
}, "mathjax-editing");

StackExchange.ready(function() {
var channelOptions = {
tags: "".split(" "),
id: "69"
};
initTagRenderer("".split(" "), "".split(" "), channelOptions);

StackExchange.using("externalEditor", function() {
// Have to fire editor after snippets, if snippets enabled
if (StackExchange.settings.snippets.snippetsEnabled) {
StackExchange.using("snippets", function() {
createEditor();
});
}
else {
createEditor();
}
});

function createEditor() {
StackExchange.prepareEditor({
heartbeatType: 'answer',
autoActivateHeartbeat: false,
convertImagesToLinks: true,
noModals: true,
showLowRepImageUploadWarning: true,
reputationToPostImages: 10,
bindNavPrevention: true,
postfix: "",
imageUploader: {
brandingHtml: "Powered by u003ca class="icon-imgur-white" href="https://imgur.com/"u003eu003c/au003e",
contentPolicyHtml: "User contributions licensed under u003ca href="https://creativecommons.org/licenses/by-sa/3.0/"u003ecc by-sa 3.0 with attribution requiredu003c/au003e u003ca href="https://stackoverflow.com/legal/content-policy"u003e(content policy)u003c/au003e",
allowUrls: true
},
noCode: true, onDemand: true,
discardSelector: ".discard-answer"
,immediatelyShowMarkdownHelp:true
});


}
});














draft saved

draft discarded


















StackExchange.ready(
function () {
StackExchange.openid.initPostLogin('.new-post-login', 'https%3a%2f%2fmath.stackexchange.com%2fquestions%2f3061211%2flaplace-transforms-second-shift-theorem%23new-answer', 'question_page');
}
);

Post as a guest















Required, but never shown

























1 Answer
1






active

oldest

votes








1 Answer
1






active

oldest

votes









active

oldest

votes






active

oldest

votes









1














If I've understood your comment correctly, then I think I see the confusion.



Recall that the second shifting theorem says that if $mathcal{L}{ f(t)} = F(s)$ then $mathcal{L}{ f(t-a)u(t-a)} = e^{-as}F(s)$



Now, let's dissect taking the Laplace transform of $frac{1}{2}t^2u(t-1)$. Note that our current function is $f(t) = frac{1}{2}t^2$. So then $f(t-1) = frac{1}{2}(t-1)^2$. As such, to get $f(t)$ in the desired form, we will conveniently add $0$ in a number of different ways:
$$f(t)=f(t) + (t-t) + (frac{1}{2}-frac{1}{2}) = frac{1}{2}(t^2-2t+1) + t - frac{1}{2} = f(t-1) + t-frac{1}{2} + (frac{1}{2}-frac{1}{2}) $$
$$= f(t-1) + (t-1) + frac{1}{2} $$
Now we have $f(t)$ in the desired form to apply the shifting theorem.






share|cite|improve this answer





















  • Thank you!! this was very helpful
    – user564900
    2 days ago
















1














If I've understood your comment correctly, then I think I see the confusion.



Recall that the second shifting theorem says that if $mathcal{L}{ f(t)} = F(s)$ then $mathcal{L}{ f(t-a)u(t-a)} = e^{-as}F(s)$



Now, let's dissect taking the Laplace transform of $frac{1}{2}t^2u(t-1)$. Note that our current function is $f(t) = frac{1}{2}t^2$. So then $f(t-1) = frac{1}{2}(t-1)^2$. As such, to get $f(t)$ in the desired form, we will conveniently add $0$ in a number of different ways:
$$f(t)=f(t) + (t-t) + (frac{1}{2}-frac{1}{2}) = frac{1}{2}(t^2-2t+1) + t - frac{1}{2} = f(t-1) + t-frac{1}{2} + (frac{1}{2}-frac{1}{2}) $$
$$= f(t-1) + (t-1) + frac{1}{2} $$
Now we have $f(t)$ in the desired form to apply the shifting theorem.






share|cite|improve this answer





















  • Thank you!! this was very helpful
    – user564900
    2 days ago














1












1








1






If I've understood your comment correctly, then I think I see the confusion.



Recall that the second shifting theorem says that if $mathcal{L}{ f(t)} = F(s)$ then $mathcal{L}{ f(t-a)u(t-a)} = e^{-as}F(s)$



Now, let's dissect taking the Laplace transform of $frac{1}{2}t^2u(t-1)$. Note that our current function is $f(t) = frac{1}{2}t^2$. So then $f(t-1) = frac{1}{2}(t-1)^2$. As such, to get $f(t)$ in the desired form, we will conveniently add $0$ in a number of different ways:
$$f(t)=f(t) + (t-t) + (frac{1}{2}-frac{1}{2}) = frac{1}{2}(t^2-2t+1) + t - frac{1}{2} = f(t-1) + t-frac{1}{2} + (frac{1}{2}-frac{1}{2}) $$
$$= f(t-1) + (t-1) + frac{1}{2} $$
Now we have $f(t)$ in the desired form to apply the shifting theorem.






share|cite|improve this answer












If I've understood your comment correctly, then I think I see the confusion.



Recall that the second shifting theorem says that if $mathcal{L}{ f(t)} = F(s)$ then $mathcal{L}{ f(t-a)u(t-a)} = e^{-as}F(s)$



Now, let's dissect taking the Laplace transform of $frac{1}{2}t^2u(t-1)$. Note that our current function is $f(t) = frac{1}{2}t^2$. So then $f(t-1) = frac{1}{2}(t-1)^2$. As such, to get $f(t)$ in the desired form, we will conveniently add $0$ in a number of different ways:
$$f(t)=f(t) + (t-t) + (frac{1}{2}-frac{1}{2}) = frac{1}{2}(t^2-2t+1) + t - frac{1}{2} = f(t-1) + t-frac{1}{2} + (frac{1}{2}-frac{1}{2}) $$
$$= f(t-1) + (t-1) + frac{1}{2} $$
Now we have $f(t)$ in the desired form to apply the shifting theorem.







share|cite|improve this answer












share|cite|improve this answer



share|cite|improve this answer










answered Jan 4 at 2:27









Theo C.

25928




25928












  • Thank you!! this was very helpful
    – user564900
    2 days ago


















  • Thank you!! this was very helpful
    – user564900
    2 days ago
















Thank you!! this was very helpful
– user564900
2 days ago




Thank you!! this was very helpful
– user564900
2 days ago


















draft saved

draft discarded




















































Thanks for contributing an answer to Mathematics Stack Exchange!


  • Please be sure to answer the question. Provide details and share your research!

But avoid



  • Asking for help, clarification, or responding to other answers.

  • Making statements based on opinion; back them up with references or personal experience.


Use MathJax to format equations. MathJax reference.


To learn more, see our tips on writing great answers.





Some of your past answers have not been well-received, and you're in danger of being blocked from answering.


Please pay close attention to the following guidance:


  • Please be sure to answer the question. Provide details and share your research!

But avoid



  • Asking for help, clarification, or responding to other answers.

  • Making statements based on opinion; back them up with references or personal experience.


To learn more, see our tips on writing great answers.




draft saved


draft discarded














StackExchange.ready(
function () {
StackExchange.openid.initPostLogin('.new-post-login', 'https%3a%2f%2fmath.stackexchange.com%2fquestions%2f3061211%2flaplace-transforms-second-shift-theorem%23new-answer', 'question_page');
}
);

Post as a guest















Required, but never shown





















































Required, but never shown














Required, but never shown












Required, but never shown







Required, but never shown

































Required, but never shown














Required, but never shown












Required, but never shown







Required, but never shown







Popular posts from this blog

An IMO inspired problem

Management

Investment